Physics Final Review Mr. Miller

Pataasin ang iyong marka sa homework at exams ngayon gamit ang Quizwiz!

you throw a basketball forward while standing on a skateboard. this causes:

backward motion of the board and you

the unit used to measure linear momentum is:

kg*m/sec

when comparing the momentum of two moving objects, which statement is ALWAYS correct?

the object with higher velocity has greater momentum

an egg dropped on the sidewalk normally breaks, but an egg dropped on the gras might not break because:

the time interval for stopping is greater when the egg hits the grass

if the direction of the velocity is west, the direction of the momentum is:

west

Determine the quantity and type of charge on an object that has 3.62 x 1012 more protons than electrons. (static electricity)

+5.8 x 10^-7 Coulombs (rounded) To determine the charge on an object, determine the number of excess protons or excess electrons. Multiply the excess by the charge of an electron or the charge of a proton - 1.6 x 10-19 C. Finally, adjust the sign of the object to + or -.

The period of the sound wave produced by a 440 Hertz tuning fork is ___________.

0.00227 seconds GIVEN: f = 440 Hz Find T T = 1 / f = 1 / (440 HZ) = 0.00227 s

Two charged objects have an attractive force of 0.080 N. If the distance separating the objects is quadrupled, then what is the new force?

0.0050 N Explanation: The electrostatic force is inversely related to the square of the separation distance. So if d is four times larger (quadrupled), then F is 16 times smaller - that is, 1/16-th the original value. One-sixteenth of 0.080 N is 0.0050 N.

Frieda the fly flaps its wings back and forth 121 times each second. The period of the wing flapping is ____ sec.

0.00826 seconds The quantity 121 times/second is the frequency. The period is the reciprocal of the frequency. T=1/(121 Hz) = 0.00826 s

Two charged objects have a repulsive force of 0.080 N. If the distance separating the objects is tripled, then what is the new force?

0.00889 N Explanation: The electrostatic force is inversely related to the square of the separation distance. So if d is three times larger, then F is nine times smaller - that is, one-ninth the original value. One-ninth of 0.080 N is 0.00889 N.

A wave is introduced into a thin wire held tight at each end. It has an amplitude of 3.8 cm, a frequency of 51.2 Hz and a distance from a crest to the neighboring trough of 12.8 cm. Determine the period of such a wave.

0.0195 sec Here is an example of a problem with a lot of extraneous information. The period is simply the reciprocal of the frequency. In this case, the period is 1/(51.2 Hz) which is 0.0195 seconds. Know your physics concepts to weed through the extra information.

Two charged objects have a repulsive force of 0.080 N. If the distance separating the objects is doubled, then what is the new force?

0.020 N Explanation: The electrostatic force is inversely related to the square of the separation distance. So if d is two times larger, then F is four times smaller - that is, one-fourth the original value. One-fourth of 0.080 N is 0.020 N.

Two charged objects have an attractive force of 0.080 N. If the charge of one of the objects is tripled and the distance separating the objects is tripled, then what is the new force?

0.0267 N Explanation: The electrostatic force is directly related to the product of the charges and inversely related to the square of the separation distance. Tripling one of the charges would serve to triple the force. Tripling the distance would serve to reduce the force by a factor of nine. The combined affect of these two variations would be to make the force 3/9-ths or 1/3-rd the original value. One-third of 0.080 N is 0.0267 N.

At what distance of separation must two 1.00-microCoulomb charges be positioned in order for the repulsive force between them to be equivalent to the weight (on Earth) of a 1.00-kg mass?

0.030 m or 3.0 cm Step 1: Identify known values in variable form. Q1 = 1.0 x 10-6 C and Q2 = 1.0 x 10-6 C Felect = Fgrav = mg = 1.0 • 9.8 m/s/s = 9.8 N Step 2: Identify requested information d = ??? Step 3: Substitute and solve. (See example C on the Tutorial page for sample algebra.)

Two charged objects have a repulsive force of 0.080 N. If the charge of one of the objects is doubled, and the distance separating the objects is doubled, then what is the new force?

0.040 N Explanation: The electrostatic force is directly related to the product of the charges and inversely related to the square of the separation distance. Doubling one of the charges would serve to double the force. Doubling the distance would serve to reduce the force by a factor of four. The combined affect of these two variations would be to decrease the force by a factor of two - changing it from 0.080 N to 0.040 N.

A positively charged object with a charge of +85 nC is being used to balance the downward force of gravity on a 1.8-gram balloon that has a charge of -63 nC. How high above the balloon must the object be held in order to balance the balloon? (NOTE: 1 nC = 1 x 10-9 C)

0.052 meters Given: Q1 = +8.5 x 10-8 C Q2 = -6.3 x 10-8 C m = 0.0018 kg Use the mass to determine the force of gravity (m • g). The force of gravity on the balloon is 0.0176 N. Thus, the upward electrical force is 0.0176 N. Now that F, Q1, and Q2 are known, Coulomb's law can be used to determine the distance d in the equation. Algebraic rearrangement leads to d = Sqrt [ (k• Q1• Q2) / F ]. Substitution leads to the answer.

Two charged objects have a repulsive force of 0.080 N. If the charge of both of the objects is doubled and the distance separating the objects is doubled, then what is the new force?

0.080 N Explanation: The electrostatic force is directly related to the product of the charges and inversely related to the square of the separation distance. Doubling both of the charges would serve to quadruple the force. Doubling the distance would serve to reduce the force by a factor of four. The combined affect of these two variations would be to not change the force at all; it remains as 0.080 N.

Two charged objects have an attractive force of 0.080 N. If the charge of one of the objects is increased by a factor of four, and the distance separating the objects is doubled, then what is the new force?

0.080 N Explanation: The electrostatic force is directly related to the product of the charges and inversely related to the square of the separation distance. Quadrupling one of the charges would serve to quadruple the force. Doubling the distance would serve to reduce the force by a factor of four. The combined affect of these two variations would be to not alter the force at all; it would remain as 0.080 N.

a hunter uses a blow gun to hunt for supper. a force of 2.0 newtons is applied to a 0.05 kg dart for 0.75 seconds. the speed of the dart as it leaves the blow gun is about:

0.13 m/sec

Joann has rubbed a balloon with wool to give it a charge of -1.0 x 10-6 C. She then acquires a plastic golf tube with a charge of +4.0 x 10-6 C localized at a given position. She holds the location of charge on the plastic golf tube a distance of 50.0 cm above the balloon. Determine the electrical force of attraction between the golf tube and the balloon.

0.144 N Step 1: Identify known values in variable form. Q1 = -1.0 x 10^-6 C and Q2 = +4.0 x 10-6 C d = 50.0 cm = 0.50 m. Step 2: Identify requested information F = ??? Step 3: Substitute and solve. (See example B on the Tutorial page for sample algebra.)

Two charged objects have a repulsive force of 0.080 N. If the charge of one of the objects is doubled, then what is the new force?

0.160 N Explanation: Electrostatic force is directly related to the charge of each object. So if the charge of one object is doubled, then the force will become two times greater. Two times 0.080 N is 0.160 N.

A balloon with a charge of 4.0 µC is held a distance of 0.70 m from a second balloon having the same charge. Calculate the magnitude of the repulsive force.

0.29 N Step 1: Identify known values in variable form. Q1 = -4.0 x 10^-6 C and Q2 = +4.0 x 10-6 C d = 0.70 m. Step 2: Identify requested information F = ??? Step 3: Substitute and solve. (See example B on the Tutorial page for sample algebra.)

Two charged objects have a repulsive force of 0.080 N. If the charge of both of the objects is doubled, then what is the new force?

0.320 N Explanation: Electrostatic force is directly related to the charge of each object. So if the charge of both objects is doubled, then the force will become four times greater. Four times 0.080 N is 0.320 N.

Two charged objects have a repulsive force of 0.080 N. If the distance separating the objects is halved, then what is the new force?

0.320 N Explanation: The electrostatic force is inversely related to the square of the separation distance. So if d is two times smaller, then F is four times larger. Four times 0.080 N is 0.320 N

A common physics lab involves the study of the oscillations of a pendulum. If a pendulum makes 33 complete back-and-forth cycles of vibration in 11 seconds, then its period is ______.

0.33 second Period refers to the time for something to happen and is measured in seconds/cycle. In this case, there are 11 seconds per 33 vibrational cycles. Thus the period is (11 s) / (33 cycles) = 0.33 seconds.

Determine the electrical force of attraction between two balloons that are charged with the opposite type of charge but the same quantity of charge. The charge on the balloons is 6.0 x 10-7 C and they are separated by a distance of 0.50 m.

1.3 x 10-2 N, attractive (rounded from 1.296 x 10-2 N) Step 1: Identify known values in variable form. Q1 = +6.0 x 10-7 C and Q2 = -6.0 x 10-7 C d = 0.50 m Step 2: Identify the requested value F = ??? Step 3: Substitute and solve (See example B on the Tutorial page for sample algebra.)

A mass is tied to a spring and begins vibrating periodically. The distance between its highest and its lowest position is 38 cm. What is the amplitude of the vibrations?

19 cm The distance that is described is the distance from the high position to the low position. The amplitude is from the middle position to either the high or the low position.

a 2 kilogram piece of clay moving at 3 m/sec strikes and sticks to a second 2 kilogram piece of clay moving at 1 m/sec in the opposite direction. Calculate the speed of the combined piece of clay.

2 m/sec

Determine the electrical force of attraction between two balloons with separate charges of +3.5 x 10-8 C and -2.9 x 10-8 C when separated a distance of 0.65 m.

2.16 x 10-5 N, attractive Step 1: Identify known values in variable form. Q1 = +3.5 x 10-8 C and Q2 = -2.9 x 10-8 C d = 0.65 m Step 2: Identify the requested value F = ??? Step 3: Substitute and solve (See example B on the Tutorial page for sample algebra.)

the time required to stop a 200 kilogram wagon moving at 5 m/sec with a 40 newton force is:

25 seconds

Balloon A and Balloon B are charged in a like manner by rubbing with animal fur. Each acquires an excess of 25 trillion electrons. If the mass of the balloons is 1 gram, then how far below Balloon B must Balloon A be held in order to levitate Balloon B? Assume the balloons act as point charges.

3.83 meters Begin by determining the Q of the balloons. Q1 = Q2 = # of excess electrons • Qelectron = 4.0 x 10-6 C. The force of gravity of the balloons is m • g or 0.0098 N. To levitate the top balloon over the bottom balloon, the electrical force of repulsion must equal the force of gravity on the top balloon. Thus Felect = 0.0098 N. Now that Q1, Q2, and F are known, Coulomb's law can be used to determine the distance d. Algebraic rearrangement leads to d = Sqrt [ (k• Q1 • Q2 ) / F ]. Substitution leads to the answer.

what is the momentum of a 30-kg cart moving at a speed of 10 meters per second?

300 kg*m/sec

The amount of charge carried by a lightning bolt is estimated at 10 Coulombs. What quantity of excess electrons is carried by the lightning bolt? (static electricity)

6.25 x 1019 electrons Multiply the charge in Coulombs (10 C) by the conversion factor: (1 electron) / (1.6 x 10-19 C). The units of C cancel; the answer is in electrons.

during a football game, a 90 kg running back moving east is tackled by a 110 kg defensive lineman running west at 5 m/sec. both players move east at 1.5 m/sec after the tackled is made. before he is tackled the running back is moving with a speed of about:

6.7 m/sec

Two 1.2-gram balloons are suspended from light strings attached to the ceiling at the same point. The net charge on the balloons is -540 nC. The balloons are distanced 68.2 cm apart when at equilibrium. Determine the length of the string.

78.8 cm Like the example problem above, it is best to begin with a sketch of the situation and a free-body diagram. Q1 and Q2 are known to be -5.4x10-7 C. The separation distance is 0.682 m. Using Coulomb's law and values of Q1, Q2, and d, the electric force can be found to be 5.64x10-3 N. The force of gravity is m • g or 0.0118 N. From the FBD and the sketch, one sees that the tangent(theta) = Felect / Fgrav = (5.64x10-3 N) / (0.0118 N) = 0.4798. Thus, theta (the angle with the vertical) is 25.6 degrees. From a distance triangle, one sees that sin(theta) = 0.5 • d / L Substituting theta and d into this equation leads to the answer.

Current has a direction. By convention, current is in the direction that ___. a. + charges move b. - electrons move c. + electrons move

A By convention, the electric current direction is the direction which positive charge would move. In wires, the actual charge carriers are negatively charged electrons. Nonetheless, the convention used for the direction of current is based on the direction which positive charges would move.

If the resistance of a circuit were tripled, then the current through the circuit would be ____. a. one-third as much b. three times as much c. unchanged d. ... nonsense! There would be no way to make such a prediction.

A Current is inversely proportional to the resistance. A threefold increase in the resistance would cause a threefold decrease in the current.

Dawn and Aram have stretched a slinky between them and begin experimenting with waves. As the frequency of the waves is doubled, a. the wavelength is halved and the speed remains constant b. the wavelength remains constant and the speed is doubled c. both the wavelength and the speed are halved. d. both the wavelength and the speed remain constant.

A Doubling the frequency will not alter the wave speed. Rather, it will halve the wavelength. Wavelength and frequency are inversely related.

A period of 5.0 seconds corresponds to a frequency of ________ Hertz. a. 0.2 b. 0.5 c. 0.02 d. 0.05 e. 0.002

A Frequency is the reciprocal of the period. The period is 5 seconds, so the frequency is 1/(5 s) = 0.20 Hz.

Olive Udadi accompanies her father to the park for an afternoon of fun. While there, she hops on the swing and begins a motion characterized by a complete back-and-forth cycle every 2 seconds. The frequency of swing is _________. a. 0.5 Hz b. 1 Hz c. 2 Hz

A Frequency refers to the number of occurrences of a periodic event per time and is measured in cycles/second. In this case, there is 1 cycle per 2 seconds. So the frequency is 1 cycles/2 s = 0.5 Hz.

Non-digital clocks (which are becoming more rare) have a second hand that rotates around in a regular and repeating fashion. The frequency of rotation of a second hand on a clock is _______ Hz. a. 1/60 b. 1/12 c. 1/2 d. 1 e. 60

A Frequency refers to the number of occurrences of a periodic event per time and is measured in cycles/second. In this case, there is 1 cycle per 60 seconds. So the frequency is f = 1 cycle / (60 s) = (1 / 60) Hz

A rubber balloon possesses a positive charge. If brought near and touched to the door of a wooden cabinet, it sticks to the door. This does not occur with an uncharged balloon. These two observations can lead one to conclude that the wall is _____. a. electrically neutral b. negatively charged c. a conductor d. lacking electrons

A Observing the positively charged balloon stick to the wall is evidence that the wall is either neutral or charged negatively. Once the second observation is made, one can conclude that the wall is neutral. If it were charged, then the wall and the uncharged balloon should attract.

As the frequency of a wave increases, the period of the wave ___________. a. decreases b. increases c. remains the same

A Period is the reciprocal of the frequency. So as f increases, 1 / f decreases.

A positively charged balloon is brought near a neutral conducting sphere as shown below. While the balloon is near, the sphere is touched (grounded). At this point, there is a movement of electrons. Electrons move ____ . a. into the sphere from the ground (hand) b. out of the sphere into the ground (hand) c. into the sphere from the balloon d. out of the sphere into the balloon e. from the ground through the sphere to the balloon f. from the balloon through the sphere to the ground g. .... nonsense! Electrons do not move at all.

A Since the balloon is not contacted to the sphere, electrons do NOT move between the balloon and the sphere (ruling out choices c, d, e, and f). The presence of the positive balloon draws electrons from ground to the sphere. This is the principle of opposites attract.

Two objects are shown at the right. One is neutral and the other is negative. Object X will ____ object Y. a. attract b. repel c. not affect

A X is charged and Y is neutral. Charged and neutral objects always attract each other.

Household circuits are often wired with two different widths of wires: 12-gauge and 14-gauge. The 12-gauge wire has a diameter of 1/12 inch while the 14-gauge wire has a diameter of 1/14 inch. Thus, 12-gauge wire has a wider cross section than 14-gauge wire. A 20-Amp circuit used for wall receptacles should be wired using 12-gauge wire and a 15-Amp circuit used for lighting and fan circuits should be wired using 14-gauge wire. Explain the physics behind such an electrical code.

A 12-gauge wire is wider than 14-gauge wire and thus has less resistance. The lesser resistance of 12-gauge wire means that it can allow charge to flow through it at a greater rate - that is, allow a larger current. Thus, 12-gauge wire is used in circuits which are protected by 20-Amp fuses and circuit breakers. On the other hand, the thinner 14-gauge wire can support less current owing to its larger resistance; it is used in circuits which are protected by 15-Amp fuses and circuit breakers.

Which of the following will cause the current through an electrical circuit to decrease? Choose all that apply. a. decrease the voltage b. decrease the resistance c. increase the voltage d. increase the resistance

A and D The current in a circuit is directly proportional to the electric potential difference impressed across the circuit and inversely proportional to the resistance of the circuit. Reducing the current can be done by reducing the voltage (choice A) or by increasing the resistance (choice D).

A circuit is wired with a power supply, a resistor and an ammeter (for measuring current). The ammeter reads a current of 24 mA (milliAmps). Determine the new current if the voltage of the power supply was ... a. ... increased by a factor of 2 and the resistance was held constant. b. ... increased by a factor of 3 and the resistance was held constant. c. ... decreased by a factor of 2 and the resistance was held constant. d. ... held constant and the resistance was increased by a factor of 2. e. ... held constant and the resistance was increased by a factor of 4. f. ... held constant and the resistance was decreased by a factor of 2. g. ... increased by a factor of 2 and the resistance was increased by a factor of 2. h. ... increased by a factor of 3 and the resistance was decreased by a factor of 2. i. ... decreased by a factor of 2 and the resistance was increased by a factor of 2.

A circuit is wired with a power supply, a resistor and an ammeter (for measuring current). The ammeter reads a current of 24 mA (milliAmps). Determine the new current if the voltage of the power supply was ... a. Inew = 48 mA (Current is directly proportional to voltage; a doubling of the voltage will double the current.) b. Inew = 72 mA (Current is directly proportional to voltage; a tripling of the voltage will triple the current.) c. Inew = 12 mA (Current is directly proportional to voltage; a halving of the voltage will halve the current.) d. Inew = 12 mA (Current is inversely proportional to resistance; a doubling of the resistance will halve the current.) e. Inew = 6 mA (Current is inversely proportional to resistance; a quadrupling of the resistance will reduce the current to one-fourth its original value.) f. Inew = 48 mA (Current is inversely proportional to resistance; a halving of the resistance will double the current.) g. Inew = 24 mA (Current is directly proportional to voltage; a doubling of the voltage will double the current. But current is also inversely proportional to the resistance; a doubling of the resistance will halve the current. These two factors offset each other and there is no overall change in the current.) h. Inew = 144 mA (Current is directly proportional to voltage; a tripling of the voltage will triple the current. But current is also inversely proportional to the resistance; a halving of the resistance will double the current. So the new current can be found by tripling and then doubling the old current of 24 mA.) i. Inew = 6 mA (Current is directly proportional to voltage; a halving of the voltage will halve the current. But current is also inversely proportional to the resistance; a doubling of the resistance will halve the current. So the new current can be found by halving and then halving again the old current of 24 mA.)

Balloons X , Y and Z are suspended from strings as shown at the right. Negatively charged balloon X attracts balloon Y and balloon Y attracts balloon Z. Balloon Z ____. List all that apply. a. may be positively charged b. may be negatively charged c. may be neutral d. must be positively charged e. must be negatively charged f. must be neutral

A, B and C Y is observed to attract a negatively charged object (balloon X). So Y could be either positively charged or neutral. Y attracts Z. If Y were neutral (and we don't know for sure that it is), then Z would attract Y if it were either + or -. So A and B are two possible answers. But Y could be positively charged. And if Y were positively charged, the Y-Z attraction would be observed if Z were neutral. So choice C is also possible.

Dizzy Smith and Hector Vector are still discussing #1e. Dizzy says that the ball is not accelerating because its velocity is not changing. Hector says that since the ball has changed its direction, there is an acceleration. Who do you agree with? Argue a position by explaining the discrepancy in the other student's argument. (Circular Motion and Satellite Motion: Acceleration)

Agree with Hector. A change in direction constitutes a velocity change and therefore an acceleration.

Mac and Tosh stand 8 meters apart and demonstrate the motion of a transverse wave on a snakey. The wave can be described as having a vertical distance of 32 cm from a trough to a crest, a frequency of 2.4 Hz, and a horizontal distance of 48 cm from a crest to the nearest trough. Determine the amplitude, period, and wavelength of such a wave.

Amplitude = 16 cm (Amplitude is the distance from the rest position to the crest position which is half the vertical distance from a trough to a crest.) Wavelength = 96 cm (Wavelength is the distance from crest to crest, which is twice the horizontal distance from crest to nearest trough.) Period = 0.42 s (The period is the reciprocal of the frequency. T = 1 / f)

Mac and Tosh stand 8 meters apart and demonstrate the motion of a transverse wave on a snakey. The wave e can be described as having a vertical distance of 32 cm from a trough to a crest, a frequency of 2.4 Hz, and a horizontal distance of 48 cm from a crest to the nearest trough. Determine the amplitude, period, and wavelength and speed of such a wave.

Amplitude = 16 cm (Amplitude is the distance from the rest position to the crest position which is half the vertical distance from a trough to a crest.) Wavelength = 96 cm (Wavelength is the distance from crest to crest, which is twice the horizontal distance from crest to nearest trough.) Period = 0.42 s (The period is the reciprocal of the frequency. T = 1 / f) Speed = 230 cm/s (The speed of a wave is calculated as the product of the frequency times the wavelength.)

TRUE or FALSE: An object that becomes grounded gains neutrons during the grounding process.

False Neutrons are positioned in the nucleus of an atom. And like protons, neutrons are never transferred in electrostatic experiments. They are bound in the nucleus and cannot escape by ordinary electrostatic methods.

Explain the connection between your answers to the above questions and the reasoning used to explain why an object moving in a circle at constant speed can be said to experience an acceleration. (Circular Motion and Satellite Motion: Acceleration)

An object which experiences either a change in the magnitude or the direction of the velocity vector can be said to be accelerating. This explains why an object moving in a circle at constant speed can be said to accelerate - the direction of the velocity changes.

Anita Diet is very concerned about her weight but seldom does anything about it. After learning about Newton's law of universal gravitation in Physics class, she becomes all concerned about the possible effect of a change in Earth's mass upon her weight. During a (rare) free moment at the lunch table, she speaks up "How would my weight change if the mass of the Earth increased by 10%?" How would you answer Anita? (Circular Motion and Satellite Motion - Lesson 3- Universal Gravitation: Newton's Law of Universal Gravitation)

Answer: "Anita - that's a great question! Since your weight is directly dependent upon the mass of the Earth, you would weigh 10% more. But don't worry honey. You wouldn't look any different than you do now since your mass would remain as is."

Anna Litical wishes to make a simple pendulum that serves as a timing device. She plans to make it such that its period is 1.00 second. What length must the pendulum have?

Answer: 24.8 cm (0.248 m) Use the equation T = 2•π•(L/g).5 Insert 1.00 s in for T and 9.8 m/s2 in for g. Give attention to your algebra: Square both sides of the equation to remove the radical. The equation becomes L = T2•g/(4•π2). The length comes out to be 0.24824 m. Round to three significant digits.

A pair of trapeze performers at the circus is swinging from ropes attached to a large elevated platform. Suppose that the performers can be treated as a simple pendulum with a length of 16 m. Determine the period for one complete back and forth cycle.

Answer: 8.0 s Use the equation T = 2•π•(L/g).5 Insert 16 m in for L and 9.8 m/s2 in for g T = 2•π•SQRT(16/9.8) = 8.028 s (round to two significant digits)

Having recently completed her first Physics course, Dawn Well has devised a new business plan based on her teacher's Physics for Better Living theme. Dawn learned that objects weigh different amounts at different distances from Earth's center. Her plan involves buying gold by the weight at one altitude and then selling it at another altitude at the same price per weight. Should Dawn buy at a high altitude and sell at a low altitude or vice versa? (Circular Motion and Satellite Motion - Lesson 3- Universal Gravitation: Newton's Law of Universal Gravitation)

Answer: Buy high and sell low The mass of the purchased gold would be the same at both altitudes. Yet it would weight less at higher altitudes. So to make a profit, Dawn should buy at high altitudes and sell at low altitudes. She would have more gold (by weight) to sell at the lower altitudes.

A medium is able to transport a wave from one location to another because the particles of the medium are ____. a. frictionless b. isolated from one another c. able to interact d. very light

Answer: C For a wave to be transmitted through a medium, the individual particles of the medium must be able to interact so that they can exert a push and/or pull on each other; this is the mechanism by which disturbances are transmitted through a medium.

Suppose that two objects attract each other with a gravitational force of 16 units. If the distance between the two objects is tripled, then what is the new force of attraction between the two objects? (Circular Motion and Satellite Motion - Lesson 3 - Universal Gravitation: The Apple, the Moon, and the Inverse Square Law)

Answer: F = 1.78 units If the distance is increased by a factor of 3, then force will be decreased by a factor of 9 (32). The new force is then 1/9 of the original 16 units. F = (16 N) / 9 = 1.78 units

Suppose that two objects attract each other with a gravitational force of 16 units. If the mass of both objects was doubled, and if the distance between the objects was doubled, then what would be the new force of attraction between the two objects? (Circular Motion and Satellite Motion - Lesson 3- Universal Gravitation: Newton's Law of Universal Gravitation)

Answer: F = 16 units If each mass is increased by a factor of 2, then force will be increased by a factor of 4 (2*2). But this affect is offset by the doubling of the distance. Doubling the distance would cause the force to be decreased by a factor of 4 (22); the result is that there is no net affect on force. F = (16 units) • 4 / 4 = 16 units

Suppose that two objects attract each other with a gravitational force of 16 units. If the mass of object 1 was doubled, and if the distance between the objects was tripled, then what would be the new force of attraction between the two objects? (Circular Motion and Satellite Motion - Lesson 3- Universal Gravitation: Newton's Law of Universal Gravitation)

Answer: F = 3.56 units If the mass of one object is doubled. then the force of attraction will be doubled as well. But this affect is more than offset by the tripling of the separation distance. Tripling the distance would cause the force to be decreased by a factor of 9 (32). The net affect on force is that it decreased by a factor of 2/9. F = (16 units) • 2 / 9 = 3.56 units

Suppose that two objects attract each other with a gravitational force of 16 units. If the mass of both objects was tripled, and if the distance between the objects was doubled, then what would be the new force of attraction between the two objects? (Circular Motion and Satellite Motion - Lesson 3- Universal Gravitation: Newton's Law of Universal Gravitation)

Answer: F = 36 units If each mass is increased by a factor of 3, then force will be increased by a factor of 9 (3*3). But this affect is partly offset by the doubling of the distance. Doubling the distance would cause the force to be decreased by a factor of 4 (22). the net affect on force is that it increased by 9/4. F = (16 units) * 9 / 4 = 36 units

Suppose that two objects attract each other with a gravitational force of 16 units. If the distance between the two objects is doubled, what is the new force of attraction between the two objects? (Circular Motion and Satellite Motion - Lesson 3 - Universal Gravitation: The Apple, the Moon, and the Inverse Square Law)

Answer: F = 4 units If the distance is increased by a factor of 2, then force will be decreased by a factor of 4 (22). The new force is then 1/4 of the original 16 units. F = (16 N) / 4 = 4 units

Suppose that two objects attract each other with a gravitational force of 16 units. If the distance between the two objects is doubled, what is the new force of attraction between the two objects? (Circular Motion and Satellite Motion - Lesson 3- Universal Gravitation: Newton's Law of Universal Gravitation)

Answer: F = 4 units If the distance is increased by a factor of 2, then force will be decreased by a factor of 4 (22). The new force is then 1/4 of the original 16 units. F = (16 units ) / 4 = 4 units

Suppose that two objects attract each other with a gravitational force of 16 units. If the distance between the two objects is reduced by a factor of 5, then what is the new force of attraction between the two objects? (Circular Motion and Satellite Motion - Lesson 3 - Universal Gravitation: The Apple, the Moon, and the Inverse Square Law)

Answer: F = 400 units If the distance is decreased by a factor of 5, then force will be increased by a factor of 25 (52). The new force is then 25 times the original 16 units. F = (16 N) • 25 = 400 units

Suppose that two objects attract each other with a gravitational force of 16 units. If the mass of both objects was doubled, and if the distance between the objects remained the same, then what would be the new force of attraction between the two objects? (Circular Motion and Satellite Motion - Lesson 3- Universal Gravitation: Newton's Law of Universal Gravitation)

Answer: F = 64 units If each mass is increased by a factor of 2, then force will be increased by a factor of 4 (2*2). The new force is then 4 times the original 16 units. F = (16 units ) • 4 = 64 units

Suppose that two objects attract each other with a gravitational force of 16 units. If the distance between the two objects is reduced in half, then what is the new force of attraction between the two objects? (Circular Motion and Satellite Motion - Lesson 3 - Universal Gravitation: The Apple, the Moon, and the Inverse Square Law)

Answer: F = 64 units If the distance is decreased by a factor of 2, then force will be increased by a factor of 4 (22). The new force is then 4 times the original 16 units. F = (16 N) • 4 = 64 units

Suppose that two objects attract each other with a gravitational force of 16 units. If the distance between the two objects is reduced in half, then what is the new force of attraction between the two objects? (Circular Motion and Satellite Motion - Lesson 3- Universal Gravitation: Newton's Law of Universal Gravitation)

Answer: F = 64 units If the distance is decreased by a factor of 2, then force will be increased by a factor of 4 (22). The new force is then 4 times the original 16 units. F = (16 units) • 4 = 64 units

Determine the centripetal force acting upon a 40-kg child who makes 10 revolutions around the Cliffhanger in 29.3 seconds. The radius of the barrel is 2.90 meters. (Circular Motion and Satellite Motion: Mathematics of Circular Motion)

Answer: Fnet = 533 Given: m = 40 kg; R = 2.90 m; T = 2.93 s (since 10 cycles takes 29.3 s). First, find speed using speed=(2 • pi • R) / T = 6.22 m/s. Then find the acceleration using a = v2 / R = = (6.22 m/s)2 / (2.90 m) = 13.3 m/s/s Now use Fnet = m • a to find that Fnet = 533 N.

As a star ages, it is believed to undergo a variety of changes. One of the last phases of a star's life is to gravitationally collapse into a black hole. What will happen to the orbit of the planets of the solar system if our star (the Sun shrinks into a black hole)? (And of course, this assumes that the planets are unaffected by prior stages of the Sun's evolving stages.) (Circular Motion and Satellite Motion - Lesson 3- Universal Gravitation: Newton's Law of Universal Gravitation)

Answer: No affect The shrinking of the sun into a black hole would not influence the amount of force with which the sun attracted the Earth since neither the mass of the sun nor the distance between the Earth's and sun's centers would change.

Which would have the highest frequency of vibration? Pendulum A: A 200-g mass attached to a 1.0-m length string Pendulum B: A 400-g mass attached to a 0.5-m length string

Answer: Pendulum B The mass of the bob is not an important variable; only the length of the string will effect the period (and thus the frequency). Frequency and period are inversely related. The pendulum with the smallest period will have the highest frequency of vibration. A longer pendulum has a higher period; a shorter pendulum will have a smaller period. Thus, the pendulum with the shorter string will have a higher frequency of vibration.

As more and more resistors are added in parallel to a circuit, the equivalent resistance of the circuit ____________ (increases, decreases) and the total current of the circuit ____________ (increases, decreases).

Answers: As more and more resistors are added in parallel to a circuit, the equivalent resistance of the circuit decreases and the total current of the circuit increases. Adding more resistors in parallel is equivalent to providing more branches through which charge can flow. Even though the added branches offer resistance to the flow of charge, the overall resistance decreases due to the fact that there are additional pathways available for charge flow. The fraction of the total charge which encounters a single resistor is now less. The additional branches mean that the circuit can sustain a greater current.

A positively charged piece of Styrofoam is placed on the table. A neutral aluminum pie plate is brought near as shown below. While held above the Styrofoam, the aluminum plate is touched (grounded). At this point, there is a movement of electrons. Electrons move ____ . a. out of the aluminum plate into the ground (hand) b. into the aluminum plate from the ground (hand) c. into the aluminum plate from the Styrofoam d. out of the aluminum plate into the Styrofoam e. from the ground through the aluminum plate to the Styrofoam f. from the Styrofoam through the aluminum plate to the ground g. .... nonsense! Electrons do not move at all.

B Since the foam is not contacted to the aluminum plate, electrons do NOT move between the foam and the aluminum (ruling out choices c, d, e, and f). The presence of the positively charged foam plate draws electrons from ground to the aluminum plate. This is the principle of opposites attract.

A pendulum bob is pulled back to position A and released from rest. The bob swings through its usual circular arc and is caught at position C. Determine the position (A, B, C or all the same) where the ... a. ... force of gravity is the greatest? b. ... restoring force is the greatest? c. ... speed is the greatest? d. ... potential energy is the greatest? e. ... kinetic energy is the greatest f. ... total mechanical energy is the greatest?

Answers: a. The force of gravity is everywhere the same since it is not dependent upon the pendulum's position; it is always the product of mass and 9.8 N/kg. b. The restoring force is greatest at A; the further that the bob is from the rest position, the greater the restoring force. c. The speed is greatest at C. The restoring force accelerates the bob from position A to position C. By the time the bob reaches C, it has accelerated to its maximum speed. d. The potential energy is the greatest at A. The potential energy is the greatest at the highest position. e. The kinetic energy is the greatest at position C; kinetic energy is greatest at the lowest position. By the time the bob reaches C, all the original potential energy has been transformed into kinetic energy. f. The total mechanical energy is everywhere the same since energy is conserved by a pendulum.

When an oil tanker car has arrived at its destination, it prepares to empty its fuel into a reservoir or tank. Part of the preparation involves connecting the body of the tanker car with a metal wire to the ground. Suggest a reason for why is this done.

As fuel is pumped from the tanker car to a reservoir, charge can quickly build up as the fluid flows through the hoses. This static charge can create sparks capable of igniting the fuel. By connecting the body of the tanker car to the ground, the static charge can be transferred to the ground. A metal wire is used since metals are conductive and allow charge to flow through them.

As the number of resistors in a series circuit increases, the overall resistance __________ (increases, decreases, remains the same) and the current in the circuit __________ (increases, decreases, remains the same).

As the number of resistors in a series circuit increases, the overall resistance increases and the current in the circuit decreases.

Based on your answers to the above question, complete the following statement: The overall or equivalent resistance of three resistors placed in parallel will be _____. a. greater than the resistance of the biggest R value of the three. b. less than the resistance of the smallest R value of the three. c. somewhere in between the smallest R and the biggest R value of the three. d. ... nonsense! No such generalization can be made. The results vary.

B An observation of the answers to question #8 reveals that in every case, the total resistance is less than the resistance of each individual resistance.

Two objects are charged as shown at the right. Object X will ____ object Y. a. attract b. repel c. not affect

B Both C and Y are charged with excess + charge. Having the same type of charge, they will repel.

If the voltage across a circuit is quadrupled, then the current through the circuit would be ____. a. one-fourth as much b. four times as much c. unchanged d. ... nonsense! There would be no way to make such a prediction.

B Current is directly proportional to the voltage. A fourfold increase in the voltage would cause a fourfold increase in the current.

If an electric circuit could be compared to a water circuit at a water park, then the current would be analogous to the ____. Choices: A. water pressure B. gallons of water flowing down slide per minute C. water D. bottom of the slide E. water pump F. top of the slide

B Current is the rate at which something flows. Electric current is the rate at which electric charge flows past a point on the electric circuit. Water current is the rate at which water flows past a point on the water circuit. As such, current is analogous to the number of gallons of water flowing into, along, and out of a slide per unit of time.

Two waves on identical strings have frequencies in a ratio of 2 to 1. If their wave speeds are the same, then how do their wavelengths compare? a. 2:1 b. 1:2 c. 4:1 d. 1:4

B Frequency and wavelength are inversely proportional to each other. The wave with the greatest frequency has the shortest wavelength. Twice the frequency means one-half the wavelength. For this reason, the wavelength ratio is the inverse of the frequency ratio.

A positively charged pop can is touched by a person standing on the ground. The pop can subsequently becomes neutral. The pop can becomes neutral during this process because ______. a. electrons pass from the pop can to the person (ground) b. electrons pass from the person (ground) to the pop can c. protons pass from the pop can to the person (ground) d. protons pass from the person (ground) to the pop can

B Protons do NOT move during electrostatic activities, so choices c and d can be ruled out. To ground a positively charged object, electrons must be added to it in order neutralize its excess positive charge. So electrons must move from the ground into the pop can.

True or False: When an object becomes polarized, it acquires a charge and becomes a charged object.

False When an object becomes polarized, its center of positive charge becomes separated from its center of negative charge. Overall, there are just as many positive charges as negative charges; the object has a balance of charges and is therefore neutral.

If a battery provides a high voltage, it can ____. a. do a lot of work over the course of its lifetime b. do a lot of work on each charge it encounters c. push a lot of charge through a circuit d. last a long time

B The electric potential difference or voltage of a battery is the potential energy difference across its terminals for every Coulomb of charge. A high voltage battery maximizes this ratio of energy/charge by doing a lot of work on each charge it encounters.

If you strike a horizontal rod vertically from above, what can be said about the waves created in the rod? a. The particles vibrate horizontally along the direction of the rod. b. The particles vibrate vertically, perpendicular to the direction of the rod. c. The particles vibrate in circles, perpendicular to the direction of the rod. d. The particles travel along the rod from the point of impact to its end.

B The particles vibrate in the direction of the source which creates the initial disturbance. Since the hammer was moving vertically, the particles will also vibrate vertically.

TRUE or FALSE? Two neutral conducting pop cans are touching each other. A negatively charged balloon is brought near Can X as shown below. As the balloon approaches Can X, there is a movement of electrons between the balloon and can X (in one direction or the other).

B - False In induction charging, there is never a transfer of electrons between the charged object (the balloon) and the object being charged (Can X). The electron movement happens between the object being charged (Can X) and the ground (Can Y). In this case, electrons would leave Can X and enter Can Y.

In the above situation, the conducting sphere is ____. List all that apply. a. charged b. uncharged (neutral) c. polarized

B and C The conducting sphere is both uncharged (there is a balance between the number of + and - charges) and polarized (there is a separation of the + from the - charges).

A child in a swing makes one complete back and forth motion in 3.2 seconds. This statement provides information about the child's a. speed b. frequency c. period

B and C We now know that the period is 3.2 seconds and that the frequency is 0.31 Hz.

A certain electrical circuit contains a battery with three cells, wires and a light bulb. Which of the following would cause the bulb to shine less brightly? Choose all that apply. a. increase the voltage of the battery (add another cell) b. decrease the voltage of the battery (remove a cell) c. decrease the resistance of the circuit d. increase the resistance of the circuit

B and D The bulb will shine less brightly if the current in it is reduced. Reducing the current can be done by reducing the electric potential difference impressed across the bulb (choice B) or by increasing the resistance of the bulb (choice D).

Which of the following materials are likely to exhibit more conductive properties than insulating properties? _____ Explain your answers. a. rubber b. aluminum c. silver d. plastic e. wet skin

B, C and E Aluminum and silver are metals, making them good conductors. The human body is a fairly good conductor. When wet, its an even better conductor.

A physics teacher rubs a glass object and a felt cloth together and the glass becomes positively charged. Which of the following statements are true? Circle all that apply. a. The glass gained protons during the rubbing process. b. The felt became charged negatively during this rubbing process. c. Charge is created during the rubbing process; it is grabbed by the more charge-hungry object. d. If the glass acquired a charge of +5 units, then the felt acquires a charge of -5 units. e. This event violates the law of conservation of charge. f. Electrons are transferred from glass to felt; protons are transferred from felt to glass. g. Once charged in this manner, the glass object and the felt cloth should attract each other. h. In general, glass materials must have a greater affinity for electrons than felt materials.

B, D, G For A: protons are not transferred in electrostatics. For C: charge is neither created nor destroyed (ever). For E: electrons are simply transferred, consistent with the law of conservation of charge. For F: protons are not transferred in electrostatic experiments (the electrons are transferred). For H: If glass transfers electrons to felt, then the felt must be more electron-hungry.

A neutral metal sphere is touched by a negatively charged metal rod. As a result, the sphere will be ____ and the metal rod will be ____. Select the two answers in their respective order. a. positively charged b. negatively charged c. neutral d. much more massive e. ... not enough information to tell

BB This is a case of charging by conduction. When a charged object is used to charge a neutral object by conduction, the previously neutral object acquires the same type of charge as the charged object. The charge object maintains the same type of charge that it originally had. So in this case, both objects have a negative charge.

A metal sphere is electrically neutral. It is touched by a positively charged metal rod. As a result, the metal sphere becomes charged positively. Which of the following occur during the process? List all that apply. a. The metal sphere gains some protons.,/p. b. Electrons are transferred from the sphere to the rod. c. The metal sphere loses electrons. d. The overall charge of the system is conserved. e. Protons are transferred from the rod to the sphere. f. Positive electrons are moved between the two objects.

BCD In electrostatic activities, protons are never transferred (which rules out choices a and e). Electrons are not positively charged (ruling out choice e). Choices B, C and D are all true and explain the essential nature of the conduction charging process.

Two waves are traveling through the same container of nitrogen gas. Wave A has a wavelength of 1.5 m. Wave B has a wavelength of 4.5 m. The speed of wave B must be ________ the speed of wave A. a. one-ninth b. one-third c. the same as d. three times larger than

C The medium is the same for both of these waves ("the same container of nitrogen gas"). Thus, the speed of the wave will be the same. Alterations in a property of a wave (such as wavelength) will not affect the speed of the wave. Two different waves travel with the same speed when present in the same medium.

3. Before beginning its initial descent, a roller coaster car is always pulled up the first hill to a high initial height. Work is done on the car (usually by a chain) to achieve this initial height. A coaster designer is considering three different incline angles at which to drag the 2000-kg car train to the top of the 60-meter high hill. In each case, the force applied to the car will be applied parallel to the hill. Her critical question is: which angle would require the most work? Analyze the data, determine the work done in each case, and answer this critical question. (Work, Energy, and Power: Calculating the Amount of Work Done by Forces ) Angle Force Distance Work (J) a. 35 deg 1.12 x 104 N 105 m Work? b. 45 deg 1.39 x 104 N 84.9 m Work? c. 55 deg 1.61 x 104 N 73.2 m Work:

Be careful! The angle in the table is the incline angle. The angle theta in the equation is the angle between F and d. If the F is parallel to the incline and the d is parallel to the incline, then the angle theta in the work equation is 0 degrees. For this reason, W=F*d*cosine 0 degrees. In each case, the work is approximately 1.18 x106 Joules. The angle does not affect the amount of work done on the roller coaster car.

Which statement best explains why a rubber rod becomes negatively charged when rubbed with fur? a. The rubber that the rod is made of is a better insulator than fur. b. The fur is a better insulator than the rubber. c. Molecules in the rubber rod have a stronger attraction for electrons than the molecules in the fur. d. Molecules in the fur have a stronger attraction for electrons than the molecules in the rubber rod.

C During a charging by rubbing (or by friction) process, the material that becomes negatively charged does so because it simply likes electrons more than the material with which it is rubbed. The conductive abilities of the two materials has nothing to do with the subsequent result of the rubbing process.

A neutral metal sphere is touched by a negatively charged metal rod. During the process, electrons are transferred from the _____ to the _____ and the sphere acquires a _____ charge. a. neutral sphere, charged rod, negative b. neutral sphere, charged rod, positive c. charged rod, neutral sphere, negative d. charged rod, neutral sphere, positive e. ... nonsense! None of these describe what occurs.

C During charging by conduction, both objects acquire the same type of charge. If a negative object is used to charge a neutral object, then both objects become charged negatively. In order for the neutral sphere to become negative, it must gain electrons from the negatively charged rod.

Electrical forces ____. (static electricity) a. can cause objects to only attract each other b. can cause objects to only repel each other c. can cause objects to attract or repel each other d. have no effect on objects

C Electric forces are repulsive for objects of like charge and attractive between objects of the opposite type of charge or between charged objects and neutral objects.

the quantity electric potential is defined as the amount of _____. a. electric potential energy b. force acting upon a charge c. potential energy per charge d. force per charge

C Electric potential is the amount of potential energy per unit of charge.

In problem #5, the period of swing is __________. a. 0.5 second b. 1 second c. 2 second

C Period refers to the time for something to happen. In this case, the period is the time for one complete swing - given as 2 seconds.

Suppose that a conducting sphere is charged positively by some method. The charge is initially deposited on the left side of the sphere. Yet because the object is conductive, the charge spreads uniformly throughout the surface of the sphere. The uniform distribution of charge is explained by the fact that ____. a. the charged atoms at the location of charge move throughout the surface of the sphere b. the excess protons move from the location of charge to the rest of the sphere c. excess electrons from the rest of the sphere are attracted towards the excess protons

C Rule out A since atoms are not capable of moving within solid spheres. Rule out B since protons are not capable of moving in electrostatic demos. C is the proper explanation since the negative electrons are attracted to the region of positive charge. The electrons migrate towards the left side of the sphere until there is a uniform distribution of positive charge.

Which of the following is not a characteristic of mechanical waves? a. They consist of disturbances or oscillations of a medium. b. They transport energy. c. They travel in a direction that is at right angles to the direction of the particles of the medium. d. They are created by a vibrating source.

C The characteristic described in statement c is a property of all transverse waves, but not necessarily of all mechanical waves. A mechanical wave can also be longitudinal.

If the electrical circuit in your Walkman were analogous to a water circuit at a water park, then the battery would be comparable to _____. a. the people that slide from the elevated positions to the ground b. the obstacles that stand in the path of the moving water c. the pump that moves water from the ground to the elevated positions d. the pipes through which water flows e. the distance that water flows through the circuit

C The electrochemical cells in an electric circuit supply the energy to pump the charge from the low energy terminal to the high energy terminal, thus providing a means by which the charge can flow. In an analogous manner, a water pump in a water park supplies the energy to pump the water from the low energy position to the high energy position. Because of the similarity between the battery in an electric circuit and a water pump in a water park, the battery is sometimes referred to as a charge pump.

Moving an electron within an electric field would change the ____ the electron. a. mass ofb. amount of charge onc. potential energy of

C When a force is required to move an electron in the direction of an electric field, its electrical potential energy increases. On the other hand, an electron moving opposite the direction of the electric field will decrease its electrical potential energy. This is because the electric field direction is in the direction which a positive charge spontaneously moves. An electron is negatively charged.

An ocean wave has an amplitude of 2.5 m. Weather conditions suddenly change such that the wave has an amplitude of 5.0 m. The amount of energy transported by the wave is __________. a. halved b. doubled c. quadrupled d. remains the same

C (quadrupled) The energy transported by a wave is directly proportional to the square of the amplitude. So whatever change occurs in the amplitude, the square of that effect impacts the energy. This means that a doubling of the amplitude results in a quadrupling of the energy. Equations are guides to thinking about how a variation in one variable affects another variable.

ean Yuss is investigating the charge on several objects and makes the following findings. object c= attracts B Object D= repels C Object E= attracts D, repels F Object F= attracts A Jean knows that object A is negatively charged and object B is electrically neutral. What can Jean Yuss definitively conclude about the charge on objects C, D, E, and F? Explain.

C and D are -; E and F are + It's best to start on the right side of the table. Observing the E-F repulsion is enough to conclude that both E and F MUST be charged with like charge. Since F and A attract, the charge on F is +; and thus the charge on E must also be +. The C-D repulsion is sufficient evidence to conclude that both C and D MUST be charged. Since D and E attract, D must have the opposite charge of E. E has been declared as +, so D MUST be -. If C has like charge as D, it must be - also.

A pendulum is observed to complete 23 full cycles in 58 seconds. Determine the period and the frequency of the pendulum.

Check your understanding answer The frequency can be thought of as the number of cycles per second. Calculating frequency involves dividing the stated number of cycles by the corresponding amount of time required to complete these cycles. In contrast, the period is the time to complete a cycle. Period is calculated by dividing the given time by the number of cycles completed in this amount of time. frequency = 23 cycles/58 seconds = 0.39655 Hz = ~0.40 Hz period = 58 seconds/23 cycles = 2.5217 sec = ~2.5 s

A current is said to exist whenever _____. a. a wire is charged b. a battery is present c. electric charges are unbalanced d. electric charges move in a loop

D Current is the rate at which charge flows. Charge will not flow in a circuit unless there is an energy source capable of creating an electric potential difference and unless there is a closed conducting loop through which the charge can move.

A tennis coach paces back and forth along the sideline 10 times in 2 minutes. The frequency of her pacing is ________ Hz. a. 5.0 b. 0.20 c. 0.12 d. 0.083

D Frequency refers to the number of occurrences of a periodic event per time and is measured in cycles/second. In this case, there are 10 cycles per 2 minutes (also known as 10 cycles per 120 seconds). So the frequency is f =10 cycles / 120 s = 0.0833 cycles/s

A sound wave is a mechanical wave, not an electromagnetic wave. This means that a. particles of the medium move perpendicular to the direction of energy transport. b. a sound wave transports its energy through a vacuum. c. particles of the medium regularly and repeatedly oscillate about their rest position. d. a medium is required in order for sound waves to transport energy.

D Mechanical waves require a medium in order to transport energy. Sound, like any mechanical wave, cannot travel through a vacuum.

Upon entering the room, you observe two balloons suspended from the ceiling. You notice that instead of hanging straight down vertically, the balloons seems to be repelling each other. You can conclusively say ... a. both balloons have a negative charge. b. both balloons have a positive charge. c. one balloon is charge positively and the other negatively. d. both balloons are charged with the same type of charge.

D Observing a repulsive interaction is sufficient evidence to conclude that both balloons are charged. However, further testing or additional information would be required to determine the type of charge the balloons have.

The drift velocity of mobile charge carriers in electric circuits is ____. a. very fast; less than but very close to the speed of light b. fast; faster than the fastest car but nowhere near the speed of light c. slow; slower than Michael Jackson runs the 220-meters d. very slow; slower than a snail

D The average speed of an electron within a circuit is very, very slow. This is due primarily to the countless collisions with the fixed atoms in the wire. Actual drift speeds depend upon numerous factors. A typical drift speed would be about 1 meter per hour.

If an electrical circuit were analogous to a water circuit at a water park, then the battery voltage would be comparable to _____. a. the rate at which water flows through the circuit b. the speed at which water flows through the circuit c. the distance that water flows through the circuit d. the water pressure between the top and bottom of the circuit e. the hindrance caused by obstacles in the path of the moving water

D The battery establishes an electric potential difference across the two ends of the external circuit and thus causes the charge to flow. The battery voltage is the numerical value of this electric potential difference. In an analogous manner, it is the difference in water pressure between the top of the water slide and the bottom of the water slide that the water pump creates. This difference in water pressure causes water to flow down the slide. Because of the similarity between electric potential difference in an electric circuit and water pressure in a water park, the quantity electric potential difference is sometimes referred to as electric pressure.

A wave is transporting energy from left to right. The particles of the medium are moving back and forth in a leftward and rightward direction. This type of wave is known as a ____. a. mechanical b. electromagnetic c. transverse d. longitudinal

D The particles are moving parallel to the direction that the wave is moving. This must be a longitudinal wave.

A transverse wave is transporting energy from east to west. The particles of the medium will move_____. a. east to west only b. both eastward and westward c. north to south only d. both northward and southward

D The particles would be moving back and forth in a direction perpendicular to energy transport. The waves are moving westward, so the particles move northward and southward.

You have likely been warned to avoid contact with electrical appliances or even electrical outlets when your hands are wet. Such contact is more dangerous when your hands are wet (vs. dry) because wet hands cause ____. a. the voltage of the circuit to be higher b. the voltage of the circuit to be lower c. your resistance to be higher d. your resistance to be lower e. the current through you to be lower

D Wet hands have less resistance and thus less hindrance to the flow of charge; the current would thus be increased. Touching an outlet with wet hands increases the risk of charge flowing through you and causing electrical shock or even electrocution.

During a physics lab, a plastic strip was rubbed with cotton and became positively charged. The correct explanation for why the plastic strip becomes positively charged is that ... a. the plastic strip acquired extra protons from the cotton. b. the plastic strip acquired extra protons during the charging process. c. protons were created as the result of the charging process. d. the plastic strip lost electrons to the cotton during the charging process.

D When two different materials are rubbed together, there is a transfer of electrons from one material to the other material. This causes one object to become positively charged (the electron loser) and the other object to become negatively charged (the electron gainer).

Two waves are traveling through a container of an inert gas. Wave A has an amplitude of 0.1 cm. Wave B has an amplitude of 0.2 cm. The energy transported by wave B must be __________ the energy transported by wave A. a. one-fourth b. one-half c. two times larger than d. four times larger than

D (four times larger) The energy transported by a wave is directly proportional to the square of the amplitude. So whatever change occurs in the amplitude, the square of that effect impacts the energy. This means that a doubling of the amplitude results in a quadrupling of the energy. Equations are guides to thinking about how a variation in one variable affects another variable.

A neutral metal sphere is touched by a positively charged metal rod. During the process, protons are transferred from the _____ to the _____ and the sphere acquires a _____ charge. a. charged rod, neutral sphere, negative b. charged rod, neutral sphere, positive c. neutral sphere, charged rod, negative d. neutral sphere, charged rod, positive e. ... nonsense! None of these describe what occurs.

E Protons are never transferred in electrostatic activities. In this case, electrons are transferred from the neutral object to the positively charged rod and the sphere becomes charged positively.

On many occasions, there is more than one force acting upon an object. A free-body diagram is a diagram that depicts the type and the direction of all the forces acting upon an object. The following descriptions and their accompanying free-body diagrams show the forces acting upon an object. For each case, indicate which force(s) are doing work upon the object. Then calculate the work done by these forces. Diagram 1: A 10-N force is applied to push a block across a friction free surface for a displacement of 5.0 m to the right. Diagram 2: A 10-N frictional force slows a moving block to a stop after a displacement of 5.0 m to the right. Diagram 3: A 10-N force is applied to push a block across a frictional surface at constant speed for a displacement of 5.0 m to the right. Diagram 4: An approximately 2-kg object is sliding at constant speed across a friction free surface for a displacement of 5 m to the right. Diagram 5: An approximately 2-kg object is pulled upward at constant speed by a 20-N force for a vertical displacement of 5 m. Work, Energy, and Power: Calculating the Amount of Work Done by Forces

Diagram 1: Forces Doing Work on the Object: Only Fapp does work. Fgrav and Fnorm do not do work since a vertical force cannot cause a horizontal displacement. Amount of Work Done by Each Force: Wapp= (10 N) * (5 m) *cos (0 degrees) = +50 Joules Diagram 2: Forces Doing Work on the Object: Only Ffrict does work. Fgrav and Fnorm do not do work since a vertical force cannot cause a horizontal displacement. Amount of Work Done by Each Force: Wfrict =(10 N) * (5 m) * cos (180 degrees) = -50 Joules Diagram 3: Forces Doing Work on the Object: Fapp and Ffrict do work. Fgrav and Fnorm do not do work since a vertical force cannot cause a horizontal displacement. Amount of Work Done by Each Force: Wapp = (10 N) * (5 m) * cos (0 deg) = +50 Joules Wfrict = (10 N) * (5 m) * cos (180 deg) = -50 Joules Diagram 4: Forces Doing Work on the Object: Neither of these forces do work. Forces do not do work when they makes a 90-degree angle with the displacement. Amount of Work Done by Each Force: No work is done. Diagram 5: Fgrav and Ftens do work. Forces do work when there is some component of force in the same or opposite direction of the displacement. Amount of Work Done by Each Force: Wtens = (20 N) * (5 m) * cos (0 deg) = +100 Joules Wgrav = (20 N) * (5 m) * cos (180 deg) = -100 Joules

Apply the work equation to determine the amount of work done by the applied force in each of the three situations described below. Diagram A: A 100N force is applied to move a 15 kg object a horizontal distance of 5 meters at constant speed. Diagram B: A 100N force is applied at an angle of 30 degrees to the horizontal to move a 15 Kg object at a constant speed for a horizontal distance of 5 m. Diagram C: An upward force is applied to lift a 15 kg object to a height of 5 meters at constant speed. Work, Energy, and Power: Calculating the Amount of Work Done by Forces

Diagram A Answer: W = (100 N) * (5 m)* cos(0 degrees) = 500 J The force and the displacement are given in the problem statement. It is said (or shown or implied) that the force and the displacement are both rightward. Since F and d are in the same direction,the angle is 0 degrees. Diagram B Answer: W = (100 N) * (5 m) * cos(30 degrees) = 433 J The force and the displacement are given in theproblem statement. It is said that the displacement is rightward. It is shown that the force is 30 degrees above the horizontal. Thus, the angle between F and d is 30 degrees. Diagram C Answer: W = (147 N) * (5 m) * cos(0 degrees) = 735 J The displacement is given in the problem statement. The applied force must be 147 N since the 15-kg mass (Fgrav=147 N) is lifted at constant speed. Since F and d are in the same direction, the angle is 0 degrees.

Rex Things and Doris Locked are out on a date. Rex makes a rapid right-hand turn. Doris begins sliding across the vinyl seat (that Rex had waxed and polished beforehand) and collides with Rex. To break the awkwardness of the situation, Rex and Doris begin discussing the physics of the motion that was just experienced. Rex suggests that objects which move in a circle experience an outward force. Thus, as the turn was made, Doris experienced an outward force that pushed her towards Rex. Doris disagrees, arguing that objects that move in a circle experience an inward force. In this case, according to Doris, Rex traveled in a circle due to the force of his door pushing him inward. Doris did not travel in a circle since there was no force pushing her inward; she merely continued in a straight line until she collided with Rex. Who is correct? Argue one of these two positions. (Circular Motion and Satellite Motion: The Centripetal Force Requirement)

Doris is correct. When the turn is made, Doris continues in a straight-line path; this is Newton's first law of motion. Once Doris collides with Rex, there is then an unbalanced force capable of accelerating Doris towards the center center of the circle, causing the circular motion.

The Cajun Cliffhanger at Great America is a ride in which occupants line the perimeter of a cylinder and spin in a circle at a high rate of turning. When the cylinder begins spinning very rapidly, the floor is removed from under the riders' feet. What affect does a doubling in speed have upon the centripetal force? Explain. (Circular Motion and Satellite Motion: Mathematics of Circular Motion)

Doubling the speed of the ride will cause the force to be four times greater than the original force. According to the equation Fnet=(m•v2) / R, force and speed2 are directly proportional. So 2X the speed means 4X the force (that's from 22).

Which of the following is true about the electrical circuit in your flashlight? a. Charge moves around the circuit very fast - nearly as fast as the speed of light. b. The battery supplies the charge (electrons) that moves through the wires. c. The battery supplies the charge (protons) that moves through the wires. d. The charge becomes used up as it passes through the light bulb. e. The battery supplies energy that raises charge from low to high voltage. f. ... nonsense! None of these are true.

E As emphasized on this page, the battery supplies the energy to move the charge through the battery, thus establishing and maintaining an electric potential difference. The battery does not supply electrons nor protons to the circuit; those are already present in the atoms of the conducting material. In fact, there would be no need to even supply charge at all since charge does not get used up in an electric circuit; only energy is used up in an electric circuit.

The Q in Coulomb's law equation stands for the _____. a. mass of a charged object b. # of excess electrons on the object c. the current of a charged object d. the distance between charged objects e. charge of a charged object

E In the equation Felect = k • Q1 • Q2 / d2 , the symbol Felect represents the electrostatic force of attraction or repulsion between objects 1 and 2. The symbol k is Coulomb's law constant (9 x 109 N • m2 / C2), Q1 and Q2 represent the quantity of charge on object 1 and object 2, and d represents the separation distance between the objects' centers.

Respond to the following student statement: (static electricity) "A positively charged object is an object that has an excess of positive electrons."

Electrons are not positively charged. Positively charged objects have an excess of protons (which are positively charged).

A conductor differs from an insulator in that a conductor ________. a. has an excess of protons b. has an excess of electrons c. can become charged and an insulator cannot d. has faster moving molecules e. does not have any neutrons to get in the way of electron flow f. none of these

F A and B are characteristic of positive and negative objects. As for C, both insulators and conductors can be charged. As for D, this has nothing to do with the conductive properties of materials. As for E, neutrons are located in the nucleus and are "out of the way" of mobile electrons.

A physics student, standing on the ground, touches an uncharged plastic baseball bat to a negatively charged electroscope. This will cause ___. a. the electroscope to be grounded as electrons flow out of the electroscope. b. the electroscope to be grounded as electrons flow into the electroscope. c. the electroscope to be grounded as protons flow out of the electroscope. d. the electroscope to be grounded as protons flow into the electroscope. e. the baseball bat to acquire an excess of protons. f. absolutely nothing (or very little) to happen since the plastic bat does not conduct.

F In order to ground an electroscope, electrons must have a conducting pathway between the ground and the object. In this case, a piece of plastic is part of the pathway connecting the ground (the student) and the charged object. Since plastic is an insulator, electrons are incapable of moving through the baseball bat. Grounding does not occur in this instance. Were there a conducting pathway available, choice a would be the proper choice.

TRUE or FALSE: Doubling the frequency of a wave source doubles the speed of the waves.

FALSE! The speed of a wave is unaffected by changes in the frequency.

TRUE or FALSE: In order for John to hear Jill, air molecules must move from the lips of Jill to the ears of John.

False. A sound wave involves the movement of energy from one location to another, not the movement of material. The air molecules are the particles of the medium, and they are only temporarily displaced, always returning to their original position.

A 1.50-kg bucket of water is tied by a rope and whirled in a circle with a radius of 1.00 m. At the top of the circular loop, the speed of the bucket is 4.00 m/s. Determine the acceleration, the net force and the individual force values when the bucket is at the top of the circular loop. (Circular Motion and Satellite Motion - Lesson 2: Newton's Second Law - Revisited)

Fgrav = m • g = 14.7 N (g is 9.8 m/s/s) a = v2 / R = (4 • Fgrav)/1 = 16 m/s/s Fnet = m • a = 1.5 kg •16 m/s/s = 24 N, down Fnet = Fgrav + Ftens, so Ftens = Fnet - Fgrav Ftens = 24 N - 14.7 N = 9.3 N

A 1.50-kg bucket of water is tied by a rope and whirled in a circle with a radius of 1.00 m. At the bottom of the circular loop, the speed of the bucket is 6.00 m/s. Determine the acceleration, the net force and the individual force values when the bucket is at the bottom of the circular loop. (Circular Motion and Satellite Motion - Lesson 2: Newton's Second Law - Revisited)

Fgrav = m • g = 14.7 N (g is 9.8 m/s/s) a = v2 / R = (62) / 1 a = 36 m/s/s Fnet = m • a = 1.5 kg • 36 m/s/s Fnet = 54 N, up Fnet = Ftens - Fgrav, so Ftens = Fnet + Fgrav Ftens = 54 N +14.7 N = 68.7 N

Noah Formula is riding a roller coaster and encounters a loop. Noah is traveling 6 m/s at the top of the loop and 18.0 m/s at the bottom of the loop. The top of the loop has a radius of curvature of 3.2 m and the bottom of the loop has a radius of curvature of 16.0 m. Use Newton's second law to determine the normal force acting upon Noah's 80-kg body at the top and at the bottom of the loop. (Circular Motion and Satellite Motion - Lesson 2: Roller Coasters and Amusement Park Physics)

Fnorm = 116 N (top) and Fnorm = 2404 N (bottom) Use a, m, and g (9.8 m/s/s) with Fnet = m • a and Fgrav = m • g to find Fnet and Fgrav. Then use a free-body diagram to find Fnorm. Top: a = v2 / R = (6 m/s)2 / (3.2 m) = 11.25 m/s2 Fnet = 900 N, down and Fgrav = 784 N, down. So Fnorm = 116 N, down. Bottom: a = v2 / R = (18 m/s)2 / (16 m) = 20.25 m/s2 Fnet = 1620 N, up and Fgrav = 784 N, So Fnorm = 2404 N, up.

Anna Litical is riding a "woody" roller coaster. Anna encounters the bottom of a small dip having a radius of curvature of 15.0 m. At the bottom of this dip Anna is traveling with a speed of 16.0 m/s and experiencing a much larger than usual normal force. Use Newton's second law to determine the normal force acting upon Anna's 50-kg body. (Circular Motion and Satellite Motion - Lesson 2: Roller Coasters and Amusement Park Physics)

Fnorm = 1343 N, up Solution is as follows: First, draw a free-body diagram and note that Fgrav = 490 N, down. Second, calculate acceleration by a = v2 / R = (16 m/s)2 / (15 m) = 17.1 m/s/s, up Then note that Fnet = m • a = 853 N, up (toward center). Now Fgrav supplies 490 N of downward force, so the Fnorm must overcome this down force and still supply the sufficient Fnet in the up direction. Therefore, Fnorm = 1343 N,up.

Anna Litical is riding on The Shock Wave at Great America. Anna experiences a downward acceleration of 12.5 m/s2 at the top of the loop and an upward acceleration of 24.0 m/s2 at the bottom of the loop. Use Newton's second law to determine the normal force acting upon Anna's 50-kg body at the top and at the bottom of the loop. (Circular Motion and Satellite Motion - Lesson 2: Roller Coasters and Amusement Park Physics)

Fnorm = 135 N (top) and Fnorm = 1690 N (bottom) Use a, m, and g (9.8 m/s/s) with Fnet = m • a and Fgrav = m•g to find Fnet and Fgrav. Then use a free-body diagram to find Fnorm. Top: Fnet = 625 N, down and Fgrav = 490 N, down. So Fnorm = 135 N, down. Bottom: Fnet =1200 N, up and Fgrav = 490 N, So Fnorm =1690 N, up.

Noah Formula is riding an old-fashioned roller coaster. Noah encounters a small hill having a radius of curvature of 12.0 m. At the crest of the hill, Noah is lifted off his seat and held in the car by the safety bar. If Noah is traveling with a speed of 14.0 m/s, then use Newton's second law to determine the force applied by the safety bar upon Noah's 80-kg body. (Circular Motion and Satellite Motion - Lesson 2: Roller Coasters and Amusement Park Physics)

Fnorm = 523 N Solution is as follows: First, draw a free-body diagram and note that Fgrav = 784 N, down. Second, calculate acceleration by a = v2 / R = (14 m/s)2 / (12 m) = 16.3 m/s/s. Then note that Fnet = m • a = 1307 N down (toward center). Now Fgrav supplies 784 N of this downward force, so the Fnorm must supply the rest. Therefore, Fnorm = 523 N.

The symbol d in Coulomb's law equation represents the distance from ___. a. A to B b. A to D c. B to C d. B to D e. C to D f. A to G g. B to F h. C to E

G In the equation Felect = k • Q1 • Q2 / d2 , the symbol Felect represents the electrostatic force of attraction or repulsion between objects 1 and 2. The symbol k is Coulomb's law constant (9 x 109 N • m2/ C2), Q1 and Q2 represent the quantity of charge on object 1 and object 2, and d represents the separation distance between the objects' centers.

Mac and Tosh are resting on top of the water near the end of the pool when Mac creates a surface wave. The wave travels the length of the pool and back in 25 seconds. The pool is 25 meters long. Determine the speed of the wave.

GIVEN: d (1-way) =25 m, t (2-way)=25 s Find v. If the pool is 25 meters long, then the back-and-forth distance is 50 meters. The wave covers this distance in 25 seconds. Now use v = d / t. v = d / t = (50 m) / (25 s) = 2 m/s

An automatic focus camera is able to focus on objects by use of an ultrasonic sound wave. The camera sends out sound waves that reflect off distant objects and return to the camera. A sensor detects the time it takes for the waves to return and then determines the distance an object is from the camera. The camera lens then focuses at that distance. Now that's a smart camera! In a subsequent life, you might have to be a camera; so try this problem for practice: If a sound wave (speed = 340 m/s) returns to the camera 0.150 seconds after leaving the camera, then how far away is the object?

GIVEN: v = 340 m/s, t = 0.150 s (down and back time) Find d (1-way) If it takes 0.150 s to travel to the object and back, then it takes 0.075 s to travel the one-way distance to the object. Now solve for time using the equation d = v • t. d = v • t = (340 m/s) • (0.075 s) = 25.5 m

While hiking through a canyon, Noah Formula lets out a scream. An echo (reflection of the scream off a nearby canyon wall) is heard 0.82 seconds after the scream. The speed of the sound wave in air is 342 m/s. Calculate the distance from Noah to the nearby canyon wall.

GIVEN: v = 342 m/s, t = 0.82 s (2-way) Find d (1-way) If it takes 0.82 s to travel to the canyon wall and back (a down-and-back time), then it takes 0.41 s to travel the one-way distance to the wall. Now use d = v • t d = v • t = (342 m/s) • (0.41 s) = 140 m

Calculate the work done by a 2.0-N force (directed at a 30° angle to the vertical) to move a 500 gram box a horizontal distance of 400 cm across a rough floor at a constant speed of 0.5 m/s. (HINT: Be cautious with the units.) Work, Energy, and Power: Calculating the Amount of Work Done by Forces

Here is a good example of the importance of understanding the angle between F and d. In this problem, the d is horizontal and the F is at a 60-degree angle to the horizontal. Thus, theta is 60 degrees. W = (2.0 N) * (4.00 m) * cos (60 degrees) = 4.0 J

If an electric circuit could be compared to a water circuit at a water park, then the ... ... battery would be analogous to the ____. ... positive terminal of the battery would be analogous to the ____. ... current would be analogous to the ____. ... charge would be analogous to the ____. ... electric potential difference would be analogous to the ____. Choices: A. water pressure B. gallons of water flowing down slide per minute C. water D. bottom of the slide E. water pump F. top of the slide

If an electric circuit could be compared to a water circuit at a water park, then the ... ... battery would be analogous to the water pump (E). ... positive terminal of the battery would be analogous to the top of the slide (F). ... current would be analogous to the gallons of water flowing down the slide per minute (B). ... charge would be analogous to the water (C). ... electric potential difference would be analogous to the water pressure (A).

If the roller coaster car in the above problem were moving with twice the speed, then what would be its new kinetic energy? Work, Energy, and Power: Kinetic Energy

If the speed is doubled, then the KE is quadrupled. Thus, KE = 4 * (1.04653 x 105 J) = 4.19 x 105 Joules. or KE = 0.5*m*v2 KE = 0.5*625 kg*(36.6 m/s)2 KE = 4.19 x 105 Joules

Anna Litical is practicing a centripetal force demonstration at home. She fills a bucket with water, ties it to a strong rope, and spins it in a circle. Anna spins the bucket when it is half-full of water and when it is quarter-full of water. In which case is more force required to spin the bucket in a circle? Explain using an equation as a "guide to thinking." (Circular Motion and Satellite Motion: Mathematics of Circular Motion)

It will require more force to accelerate a full bucket of water compared to a half-full bucket. According to the equation Fnet = m•v2 / R, force and mass are directly proportional. So the greater the mass, the greater the force.

Missy Diwater, the former platform diver for the Ringling Brother's Circus, had a kinetic energy of 12 000 J just prior to hitting the bucket of water. If Missy's mass is 40 kg, then what is her speed? Work, Energy, and Power: Kinetic Energy

KE = 0.5*m*v2 12 000 J = (0.5) * (40 kg) * v2 300 J = (0.5) * v2 600 J = v2 v = 24.5 m/s

Determine the kinetic energy of a 625-kg roller coaster car that is moving with a speed of 18.3 m/s. Work, Energy, and Power: Kinetic Energy

KE = 0.5*m*v2 KE = (0.5) * (625 kg) * (18.3 m/s)2 KE = 1.05 x105 Joules

A 900-kg compact car moving at 60 mi/hr has approximately 320 000 Joules of kinetic energy. Estimate its new kinetic energy if it is moving at 30 mi/hr. (HINT: use the kinetic energy equation as a "guide to thinking.") Work, Energy, and Power: Kinetic Energy

KE = 80 000 J The KE is directly related to the square of the speed. If the speed is reduced by a factor of 2 (as in from 60 mi/hr to 30 mi/hr) then the KE will be reduced by a factor of 4. Thus, the new KE is (320 000 J)/4 or 80 000 J.

Complete the following statements: (static electricity) After some rather exhausting counting (and a rather tall tale), a physics teacher determines that a very small sample of an object contains ... a. ... 8.25749 x 1017 protons and 5.26 x 1014 electrons; the charge on this object is ____ Coulombs. b. ... 3.12 x 1014 protons and 4.5488 x 1016 electrons; the charge on this object is ____ Coulombs. c. ... 2.40277 x 1019 protons and 9.88 x 1016 electrons; the charge on this object is ____ Coulombs. d. ... 2.6325 x 1015 protons and 2.6325 x 1015 electrons; the charge on this object is ____ Coulombs.

Method: Subtract the smaller number from the larger number. (This would be based upon the exponent.) Then multiply the difference by the charge of a proton or electron - 1.6 x 10-19 C. Answers: a. 0.132 C b. 0.00723 C (7.23 x 10-3 C) c. 3.83 C d. 0 Coulombs

Minute after minute, hour after hour, day after day, ocean waves continue to splash onto the shore. Explain why the beach is not completely submerged and why the middle of the ocean has not yet been depleted of its water supply.

Ocean waves do not transport water. An ocean wave could not bring a single drop of water from the middle of the ocean to shore. Ocean waves can only bring energy to the shore; the particles of the medium (water) simply oscillate about their fixed position. As such, water does not pile up on the beach.

A cart is loaded with a brick and pulled at constant speed along an inclined plane to the height of a seat-top. If the mass of the loaded cart is 3.0 kg and the height of the seat top is 0.45 meters, then what is the potential energy of the loaded cart at the height of the seat-top? Work, Energy, and Power: Potential Energy

PE = m*g*h PE = (3 kg ) * (9.8 m/s/s) * (0.45 m) PE = 13.2 J

Saran Wrap has a larger electron affinity than Nylon. If Nylon is rubbed against Saran Wrap, which would end up with the excess negative charge? ____________ Explain.

Saran Wrap When two materials are rubbed together, the material with the greatest affinity for electrons is the material which takes electrons away from the other material. Saran wrap takes electrons from nylon and acquires the negative charge. In turn, the nylon loses electrons and becomes charged positively.

Friction- Newton's Laws Lesson 2 Types of Forces- Sliding versus Static Friction:

Sliding friction results when an object slides across a surface. As an example, consider pushing a box across a floor. The floor surface offers resistance to the movement of the box.

Friction- Newton's Laws Lesson 2 Types of Forces- Sliding versus Static Friction:

Static friction results when the surfaces of two objects are at rest relative to one another and a force exists on one of the objects to set it into motion relative to the other object.

Kara Lott is practicing winter driving in the GBS parking lot. Kara turns the wheel to make a left-hand turn but her car continues in a straight line across the ice. Teacher A and Teacher B had viewed the phenomenon. Teacher A argues that the lack of a frictional force between the tires and the ice results in a balance of forces that keeps the car traveling in a straight line. Teacher B argues that the ice placed an outward force on the tire to balance the turning force and thus keep the car traveling in a straight line. Which teacher is (A or B) is the physics teacher? ______ Explain the fallacy in the other teacher's argument. (Circular Motion and Satellite Motion: The Centripetal Force Requirement)

Teacher A is correct (and is hopefully the physics teacher). A car turns in a circle due to the friction against its turned wheels. With wheels turned and no friction, there would be no circle. That is the problem in this situation.

Identify the three controls on an automobile that allow the car to be accelerated. (Circular Motion and Satellite Motion: Acceleration)

The accelerator allows the car to increase speed. The brake pedal allows the car to decrease the speed. And the steering wheel allows the car to change direction.

When comparing mass and size data for the planets Earth and Jupiter, it is observed that Jupiter is about 300 times more massive than Earth. One might quickly conclude that an object on the surface of Jupiter would weigh 300 times more than on the surface of the Earth. For instance, one might expect a person who weighs 500 N on Earth would weigh 150000 N on the surface of Jupiter. Yet this is not the case. In fact, a 500-N person on Earth weighs about 1500 N on the surface of Jupiter. Explain how this can be. (Circular Motion and Satellite Motion - Lesson 3- Universal Gravitation: Newton's Law of Universal Gravitation)

The affect of the greater mass of Jupiter is partly offset by the fact that the radius of Jupiter is larger. An object on Jupiter's surface is 10 times farther from Jupiter's center than it would be if on Earth's surface. So the 300-fold increase in force (due to the greater mass) must be divided by 100 since the separation distance is 10 times greater.

A Lincoln Continental and a Yugo are making a turn. The Lincoln is four times more massive than the Yugo. If they make the turn at the same speed, then how do the centripetal forces acting upon the two cars compare. Explain. (Circular Motion and Satellite Motion: Mathematics of Circular Motion)

The centripetal force on the Continental is four times greater than that of a Yugo. According to the equation Fnet=(m•v2) / R, force and mass are directly proportional. So 4 times the mass means 4 times the force.

Friction- Newton's Laws Lesson 2 Types of Forces- Sliding versus Static Friction:

The formula is: Ffrict-sliding = μfrict-sliding • Fnorm

Ben Travlun carries a 200-N suitcase up three flights of stairs (a height of 10.0 m) and then pushes it with a horizontal force of 50.0 N at a constant speed of 0.5 m/s for a horizontal distance of 35.0 meters. How much work does Ben do on his suitcase during this entire motion? Work, Energy, and Power: Calculating the Amount of Work Done by Forces

The motion has two parts: pulling vertically to displace the suitcase vertically (angle = 0 degrees) and pushing horizontally to displace the suitcase horizontally (angle = 0 degrees). For the vertical part, W = (200 N) * (10 m) * cos (0 deg) = 2000 J. For the horizontal part, W = (50 N) * (35 m) * cos (0 deg) = 1750 J. The total work done is 3750 J (the sum of the two parts).

Mac and Tosh are experimenting with pulses on a rope. They vibrate an end up and down to create the pulse and observe it moving from end to end. How does the position of a point on the rope, before the pulse comes, compare to the position after the pulse has passed?

The point returns to its original position. Waves (and pulses) do not permanently displace particles from their rest position.

A tired squirrel (mass of 1 kg) does push-ups by applying a force to elevate its center-of-mass by 5 cm. Estimate the number of push-ups that a tired squirrel must do in order to do a approximately 5.0 Joules of work. Work, Energy, and Power: Calculating the Amount of Work Done by Forces

The squirrel applies a force of approximately 10 N (9.8 N to be exact) to raise its body at constant speed. The displacement is 0.05 meters. The angle between the upward force and the upward displacement is 0 degrees. The work for 1 push-up is approximately W = 10 N * 0.05 m * cos 0 degrees = 0.5 Joules If the squirrel does a total of 5.0 Joules of work, then it must have done about 10 push-ups.

A student with a mass of 80.0 kg runs up three flights of stairs in 12.0 sec. The student has gone a vertical distance of 8.0 m. Determine the amount of work done by the student to elevate his body to this height. Assume that his speed is constant. Work, Energy, and Power: Calculating the Amount of Work Done by Forces

The student weighs 784 N (Fgrav= 80 kg * 9.8 m/s/s). To lift a 784-Newton person at constant speed, 784 N of force must be applied to it (Newton's laws). The force is up, the displacement is up, and so the angle theta in the work equation is 0 degrees. Thus, W = (784 N) * (8 m) * cos (0 degrees) = 6272 Joules

Friction- Newton's Laws Lesson 2 Types of Forces- Sliding versus Static Friction:

The symbol μfrict-sliding represents the coefficient of sliding friction between the two surfaces. The coefficient value is dependent primarily upon the nature of the surfaces that are in contact with each other.

Friction- Newton's Laws Lesson 2 Types of Forces- Sliding versus Static Friction:

The values of μ provide a measure of the relative amount of adhesion or attraction of the two surfaces for each other.

Ocean waves are observed to travel along the water surface during a developing storm. A Coast Guard weather station observes that there is a vertical distance from high point to low point of 4.6 meters and a horizontal distance of 8.6 meters between adjacent crests. The waves splash into the station once every 6.2 seconds. Determine the frequency and the speed of these waves.

The wavelength is 8.6 meters and the period is 6.2 seconds. The frequency can be determined from the period. If T = 6.2 s, then f =1 /T = 1 / (6.2 s) f = 0.161 Hz Now find speed using the v = f • λ equation. v = f • λ = (0.161 Hz) • (8.6 m) v = 1.4 m/s

Having recently completed his first Physics course, Noah Formula has devised a new business plan based on his teacher's Physics for Better Living theme. Noah learned that objects weigh different amounts at different distances from Earth's center. His plan involves buying gold by the weight at one altitude and then selling it at another altitude at the same price per weight. Should Noah buy at a high altitude and sell at a low altitude or vice versa? (Circular Motion and Satellite Motion - Lesson 3 - Universal Gravitation: The Apple, the Moon, and the Inverse Square Law)

To profit, buy at a high altitude and sell at a low one. Gold will weigh less at a high altitude and so you will get more gold for your money by buying at the high altitude. Then sell at a low altitude where the gold will weigh more than it did where it was purchased. This illustrates the inverse relationship between force of gravity (a.k.a. "weight") and distance from Earth's center.

Use the Ohm's law equation to provide numerical answers to the following questions: a. An electrical device with a resistance of 3.0 Ω will allow a current of 4.0 amps to flow through it if a voltage drop of ________ Volts is impressed across the device. b. When a voltage of 120 V is impressed across an electric heater, a current of 10.0 amps will flow through the heater if the resistance is ________ Ω. c. A flashlight that is powered by 3 Volts and uses a bulb with a resistance of 60 Ω will have a current of ________ Amps.

Use the equation V = I • R to solve for the unknown quantity. a. An electrical device with a resistance of 3.0 Ω will allow a current of 4.0 amps to flow through it if a voltage drop of 12 Volts is impressed across the device. b. When a voltage of 120 V is impressed across an electric heater, a current of 10.0 amps will flow through the heater if the resistance is 12 Ω. c. A flashlight that is powered by 3 Volts and uses a bulb with a resistance of 60 Ω will have a current of 0.05 Amps.

Use your understanding of equivalent resistance to complete the following statements: a. Two 3-Ω resistors placed in series would provide a resistance that is equivalent to one _____-Ω resistor. b. Three 3-Ω resistors placed in series would provide a resistance that is equivalent to one _____-Ω resistor. c. Three 5-Ω resistors placed in series would provide a resistance that is equivalent to one _____-Ω resistor. d. Three resistors with resistance values of 2-Ω , 4-Ω , and 6-Ω are placed in series. These would provide a resistance that is equivalent to one _____-Ω resistor. e. Three resistors with resistance values of 5-Ω , 6-Ω , and 7-Ω are placed in series. These would provide a resistance that is equivalent to one _____-Ω resistor. f. Three resistors with resistance values of 12-Ω, 3-Ω, and 21-Ω are placed in series. These would provide a resistance that is equivalent to one _____-Ω resistor.

Use the equation for the equivalent resistance of a series circuit: Req = R1 + R2 + R3 + ... a. Two 3-Ω resistors placed in series would provide a resistance which is equivalent to one 6-Ω resistor. b. Three 3-Ω resistors placed in series would provide a resistance which is equivalent to one 9-Ω resistor. c. Three 5-Ω resistors placed in series would provide a resistance which is equivalent to one 15-Ω resistor. d. Three resistors with resistance values of 2-Ω , 4-Ω , and 6-Ω are placed in series. These would provide a resistance which is equivalent to one 12-Ω resistor. e. Three resistors with resistance values of 5-Ω , 6-Ω , and 7-Ω are placed in series. These would provide a resistance which is equivalent to one 18-Ω resistor. f. Three resistors with resistance values of 12-Ω , 3-Ω , and 21-Ω are placed in series. These would provide a resistance which is equivalent to one 36-Ω resistor.

Friction- Newton's Laws Lesson 2 Types of Forces- Sliding versus Static Friction:

Values of μ sliding have been experimentally determined for a variety of surface combinations and are often tabulated in technical manuals and handbooks.

If a force of 14.7 N is used to drag the loaded cart (from previous question) along the incline for a distance of 0.90 meters, then how much work is done on the loaded cart? Work, Energy, and Power: Potential Energy

W = F * d * cos Theta W = 14.7 N * 0.9 m * cos (0 degrees) W = 13.2 J

A force of 50 N acts on the block at the angle shown in the diagram. The block moves a horizontal distance of 3.0 m. How much work is done by the applied force? (30 degrees) Work, Energy, and Power: Calculating the Amount of Work Done by Forces

W = F * d * cos(Theta) W = (50 N) * (3 m) * cos (30 degrees) = 129.9 Joules

Curly and Moe are conducting a wave experiment using a slinky. Curly introduces a disturbance into the slinky by giving it a quick back and forth jerk. Moe places his cheek (facial) at the opposite end of the slinky. Using the terminology of this unit, describe what Moe experiences as the pulse reaches the other end of the slinky. See Answer

When the slinky reaches the end of the slinky and hits Moe in the cheek, Moe expereinces a pulse of energy. The energy originated on Curly's end and is transported through the medium to Moe's end. The last particle on Moe's end transports that energy to Moe's cheek.

Complete the following statement: When work is done on a positive test charge by an external force to move it from one location to another, potential energy _________ (increases, decreases) and electric potential _________ (increases, decreases).

When work is done on a positive test charge to move it from one location to another, potential energy increases and electric potential increases.

In the movie Tango and Cash, Kurt Russell and Sylvester Stallone escape from a prison by jumping off the top of a tall wall through the air and onto a high-voltage power line. Before the jump, Stallone objects to the idea, telling Russell "We're going to fry." Russell responds with "You didn't take high school Physics did you. As long as you're only touching one wire and you're feet aren't touching the ground, you don't get electrocuted." Is this a correct statement?

Yes! In order for there to be a sustained flow of charge from one location to another, there must be a difference in electric potential. In this case, there would be a momentary flow of charge between the wire and the actor until they reach the same electric potential. Once at the same potential, charge flow would cease and there would be no electrocution. However, if the actor's feet touched the ground (an electric potential of 0) or another wire of a different potential, then there would be a sustained charge flow which likely would lead to electrocution.

. In the hammer throw, a sphere is whirled around in a circular path on the end of a chain. After revolving about five times the thrower releases his grip on the chain and the "hammer" is launched at an angle to the horizontal. A diagram of the athlete and the hammer is shown to the right. Assume that the hammer is moving in a circle in a horizontal plane with a speed of 27.0 m/s. Assume that the hammer has a mass of 7.30-kg and that it moves in a circle with a 1.25-m radius. Since the hammer is moving in a horizontal plane, the centripetal force is directed horizontally. The vertical component of the tension in the chain (directed upward) is balanced by the weight of the hammer (directed downward). Use the diagram and an understanding of vector components to determine the tension in the chain. (Circular Motion and Satellite Motion - Lesson 2: Athletics)

a = v2 / R = (27.0 m/s)2 / (1.25 m) = 583 m/s/s Fnet = m • a = (7.30 kg) • (583 m/s/s) = 4257 N Fgrav = Fvert = m • g = 71.5 N Fhoriz = Fnet = 4257 N Ftens = SQRT(Fvert2 + Fhoriz2) Ftens = 4258 N

A 55.0-kg softball player runs at 7.0 m/s around a curve whose radius is 15.0 m. The contact force (vector combination of the frictional force and the normal force) acting between the ground and the player's feet supply both the centripetal force for making the turn and the upward force for balancing the player's weight. Use a free-body diagram and your understanding of circular motion and Newton's second law to determine: a. acceleration b. Fgrav c. Fnorm d. Ffrict e. Angle of lean (Circular Motion and Satellite Motion - Lesson 2: Athletics)

a = v2 / R = (7.0 m/s)2 / (15.0 m) = 3.27 m/s/s Fnet = m • a = (55.0 kg) • (3.27 m/s/s) = 180 N Fgrav = Fvert = m • g = 539 N Fhoriz = Fnet = Ffrict = 180 N Theta = invtan(Fvert / Fhoriz) = invtan( 539 N / 180 N ) Theta = 71.6 degrees

which of the following has the greatest momentum? a tractor with a mass of 2350 kg moving at a velocity of 26 m/sec? a boat with a mass of 1310 kg moving at a velocity of 53 m/sec? a truck with a mass of 6220 kg moving at a velocity of 11 m/sec? a car with a mass of 1440 kg moving at a velocity of 39 m/sec?

a boat with a mass of 1310 kg moving at a velocity of 53 m/sec?

a collision between two object referred to as elastic would be characterized by:

a loss of kinetic energy and no permanent change in shape

Use your understanding of the mathematical relationship between work, potential energy, charge and electric potential difference to complete the following statements: a. A 9-volt battery will increase the potential energy of 1 coulomb of charge by ____ joules. b. A 9-volt battery will increase the potential energy of 2 coulombs of charge by ____ joules. c. A 9-volt battery will increase the potential energy of 0.5 coulombs of charge by ____ joules. d. A ___-volt battery will increase the potential energy of 3 coulombs of charge by 18 joules. e. A ___-volt battery will increase the potential energy of 2 coulombs of charge by 3 joules. f. A 1.5-volt battery will increase the potential energy of ____ coulombs of charge by 0.75 joules. g. A 12-volt battery will increase the potential energy of ____ coulombs of charge by 6 joules.

a. A 9-Volt battery will increase the potential energy of 1 Coulomb of charge by 9 Joules. b. A 9-Volt battery will increase the potential energy of 2 Coulombs of charge by 18 Joules. c. A 9-Volt battery will increase the potential energy of 0.5 Coulombs of charge by 4.5 Joules. d. A 6 -Volt battery will increase the potential energy of 3 Coulombs of charge by 18 Joules. e. A 1.5 -Volt battery will increase the potential energy of 2 Coulombs of charge by 3 Joules. f. A 1.5 Volt battery will increase the potential energy of 0.5 Coulombs of charge by 0.75 Joules. g. A 12 Volt battery will increase the potential energy of 0.5 Coulombs of charge by 6 Joules.

Identify the following particles as being charged or uncharged. If charged, indicate whether they are charged positively or negatively. (n = neutron, p = proton, e = electron) (static electricity) Particle A= 8e outside 2e inside 10 neutrons 9 protons Particle B= 1 Electrons second outer layer 8 electrons outer layer 2 electrons inside 12 neutrons inside 11 protons Particle C= 8 electrons second outer layer 8 electrons outer layer 2 electrons inside 20 neutrons inside 20 protons inside

a. Charged Negatively There are 11 electrons and 10 protons. This results in an imbalance of charge. With more electrons than protons, the particle is negatively charged. b. Uncharged There are 11 electrons and 11 protons. This results in a balance of charge. This particle is neutral or uncharged. c. Charged Positively There are 18 electrons and 20 protons. This results in an imbalance of charge. With more protons than electrons, the particele is positively charged.

Two neutral conducting pop cans are touching each other. A positively charged balloon is brought near one of the cans as shown below. The cans are separated while the balloon is nearby, as shown. After the balloon is removed the cans are brought back together. When touching again, can X is ____. a. positively charged b. negatively charged c. neutral d. impossible to tell

c. Neutral When the balloon is near, electrons leave Can Y and enter Can X. Overall, the two cans are neutral; yet as individual cans, Can X is negatively charged and Can Y is positively charged. When the cans are touched again, the excess electrons in Can Y return to Can X. Once more, the overall charge on the system of two cans is zero - the system is neutral.

____ are the charged parts of an atom. (Static Electricity) . Only electrons b. Only protons c. Neutrons only d. Electrons and neutrons e. Electrons and protons f. Protons and neutrons

electrons and protons

A ruby-throated hummingbird beats its wings at a rate of about 70 wing beats per second. a. What is the frequency in Hertz of the sound wave? b. Assuming the sound wave moves with a velocity of 350 m/s, what is the wavelength of the wave?

f = 70 Hz and λ = 5.0 m The frequency is given and the wavelength is the v/f ratio.

TRUE or FALSE: An object that is electrically neutral contains only neutrons. (static electricity)

false

TRUE or FALSE: An object that is negatively charged could contain only electrons with no accompanying protons. (static electricity)

false

TRUE or FALSE: An object that is positively charged contains all protons and no electrons. (static electricity)

false

Two neutral conducting pop cans are touching each other. A positively charged glass rod is brought near Can X as shown below. Which of the following occur as the glass rod approaches Can X? List all that apply. a. Electrons jump from the glass rod to can X. b. Electrons jump from the glass rod to can Y. c. Electrons jump from can X to the glass rod. d. Electrons jump from can Y to the glass rod. e. Protons jump from the glass rod to can X. f. Protons jump from can X to the glass rod. g. ... nonsense! None of these occur.

g Since contact is not made between the glass rod and Can X, there is no transfer of electrons between them. And of course, there is never a transfer of protons in electrostatic experiments. The glass rod simply induces the movement of electrons from Can Y to Can X, causing Can X to acquire a negative charge and Can Y to acquire a positive charge.

Use your understanding of equivalent resistance to complete the following statements: a. Two 6-Ω resistors placed in parallel would provide a resistance that is equivalent to one _____-Ω resistor. b. Three 6-Ω resistors placed in parallel would provide a resistance that is equivalent to one _____-Ω resistor. c. Three 8-Ω resistors placed in parallel would provide a resistance that is equivalent to one _____-Ω resistor. d. Three resistors with resistance values of 2-Ω, 4-Ω, and 6-Ω are placed in parallel. These would provide a resistance that is equivalent to one _____-Ω resistor. e. Three resistors with resistance values of 5-Ω, 6-Ω, and 7-Ω are placed in parallel. These would provide a resistance that is equivalent to one _____-Ω resistor. f. Three resistors with resistance values of 12-Ω, 6-Ω, and 21-Ω are placed in parallel. These would provide a resistance that is equivalent to one _____-Ω resistor.

in each case, use the equation 1/Req = 1/R1 + 1/R2 + 1/R3 to calculate the equivalent resistance. Answers: a. Two 6-Ω resistors placed in parallel would provide a resistance which is equivalent to one 3-Ω resistor. b. Three 6-Ω resistors placed in parallel would provide a resistance which is equivalent to one 2-Ω resistor. c. Three 8-Ω resistors placed in parallel would provide a resistance which is equivalent to one 2.67-Ω resistor. d. Three resistors with resistance values of 2-Ω , 4-Ω , and 6-Ω are placed in parallel. These would provide a resistance which is equivalent to one 1.09-Ω resistor. e. Three resistors with resistance values of 5-Ω , 6-Ω , and 7-Ω are placed in parallel. These would provide a resistance which is equivalent to one 1.96-Ω resistor. f. Three resistors with resistance values of 12-Ω , 6-Ω , and 21-Ω are placed in parallel. These would provide a resistance which is equivalent to one 3.36-Ω resistor.

a skateboarder increases her speed from 2 m/sec to 4 m/sec. her momentum:

increases

air bags help reduce injury in automobile accidents by:

increasing the time that force is applied

Suppose that you have a mass of 70 kg (equivalent to a 154-pound person). How much mass must another object have in order for your body and the other object to attract each other with a force of 1-Newton when separated by 10 meters? (Circular Motion and Satellite Motion - Lesson 3 - Universal Gravitation: Cavendish and the Value of G)

m = 2.14 x 1010 kg Use the equation Fgrav = G • m1 • m2 / d2 where m1 = 70 kg, d = 10 m and G = 6.673 x 10-11 N•m2/kg2. Substitute and solve for m2. Note that the object is equivalent to an approximately 23 million ton object!! It takes a large mass to have a significant gravitational force.

if the speed of an object is doubled, which of the following must also double?

momentum

metal sphere A has a mass of 5 kg, and a metal sphere B has a mass of 9 kilograms. the metal spheres are dropped from a 4 story window at the same time. as they reach the ground simultanously, metal sphere B has a larger:

momentum

How much work is done by an applied force to lift a 15-Newton block 3.0 meters vertically at a constant speed? Work, Energy, and Power: Calculating the Amount of Work Done by Forces

o lift a 15-Newton block at constant speed, 15-N of force must be applied to it (Newton's laws). Thus, W = (15 N) * (3 m) * cos (0 degrees) = 45 Joules

a billiard ball collides with a second ball and stops. the total momentum of the billiard balls:

remains the same

a force must be applied to stop a moving wagon. increasing the time interval over which the force is applied:

requieres a smaller force


Kaugnay na mga set ng pag-aaral

Intro to Construction Drawings-Part 3

View Set

Greek and Roman mythology in the Harry Potter books Greek

View Set

Success in College and Life - Website Evaluation

View Set

N524 EAQ #6 Fundamentals Of Nursing: Fundamental Skills

View Set

Growth and Development peds final test

View Set

ATI Mobility & Tissue Integrity Quiz

View Set

Real Estate Appraisal: Income Approach to Value Part 2 Selecting an Overall Cap Rate

View Set